16
Clinical Perspectives in Hyponatremia MARCH 2012 Faculty Editor’s Note As a physician scientist who has been studying and treating hyponatremic patients for the past 30 years, I am pleased to introduce this case-based continuing medical education publication and associated Web-based interactive learning program, Clinical Perspectives in Hyponatremia, to my colleagues in cardiology, endocrinology, critical care, nephrology, hepatology, and hospital-based internal medicine. This publication presents an overview on the pathophysiology and significance of hyponatremia and 4 case studies. Additional cases are available at www.hyponatremiaCME.org. While hyponatremia is the most common disorder of fluid and electrolyte balance seen in clinical practice, it is not easily recognized and remains underdiagnosed. With a wide range of underlying diseases and causes including heart failure, cirrhosis, and the syndrome of inappropriate antidiuretic hormone secretion (SIADH), diagnosis and management can present challenging clinical issues and decisions. Informed decision making has never been more critical as new pharmacotherapeutics continue to widen treatment possibilities. Thus, we hope Clinical Perspectives in Hyponatremia will expand your understanding of the science behind hypoantremia and the therapeutic options available to you in your management of this challenging disorder. Sincerely, Joseph G. Verbalis, MD Professor of Medicine and Physiology Chief, Division of Endocrinology and Metabolism Georgetown University Washington, DC Contents Continuing Medical Education Information .............................................................................. 2 The Pathophysiology and Significance of Hyponatremia ...................................................... 3 Case Study 1: Hyponatremia in SIADH ...................................................................................... 8 Case Study 2: Hyponatremia in Heart Failure ........................................................................ 10 Case Study 3: Hyponatremia in Neurocritical Care .............................................................. 12 Case Study 4: Hyponatremia in Cirrhosis ................................................................................ 13 Posttest ......................................................................................................................................... 15 Evaluation Form ........................................................................................................................... 16 Joseph G. Verbalis, MD Editor Paul J. Hauptman, MD Stephan A. Mayer, MD, FCCM Florence Wong, MBBS, MD, FRACP, FRCPC Visit www.HyponatremiaCME.org for additional cases and activities

Visit Clinical Perspectives in ... · Clinical Perspectives in Hyponatremia n March 2012 3 The PaThoPhysiology and significance of hyPonaTremia JosePh g. Verbalis, md Clinical Perspectives

  • Upload
    others

  • View
    3

  • Download
    0

Embed Size (px)

Citation preview

Page 1: Visit Clinical Perspectives in ... · Clinical Perspectives in Hyponatremia n March 2012 3 The PaThoPhysiology and significance of hyPonaTremia JosePh g. Verbalis, md Clinical Perspectives

Clinical Perspectives in Hyponatremia™

MARCH 2012

FacultyEditor’s Note

As a physician scientist who has been studying and treating hyponatremic patients for the past 30 years, I am pleased to introduce this case-based continuing medical education publication and associated Web-based interactive learning program, Clinical Perspectives in Hyponatremia, to my colleagues in cardiology, endocrinology, critical care, nephrology, hepatology, and hospital-based internal medicine. This publication presents an overview on the pathophysiology and significance of hyponatremia and 4 case studies. Additional cases are available at www.hyponatremiaCME.org.

While hyponatremia is the most common disorder of fluid and electrolyte balance seen in clinical practice, it is not easily recognized and remains underdiagnosed. With a wide range of underlying diseases and causes including heart failure, cirrhosis, and the syndrome of inappropriate antidiuretic hormone secretion (SIADH), diagnosis and management can present challenging clinical issues and decisions. Informed decision making has never been more critical as new pharmacotherapeutics continue to widen treatment possibilities. Thus, we hope Clinical Perspectives in Hyponatremia will expand your understanding of the science behind hypoantremia and the therapeutic options available to you in your management of this challenging disorder.

Sincerely,

Joseph G. Verbalis, MDProfessor of Medicine and PhysiologyChief, Division of Endocrinology and MetabolismGeorgetown UniversityWashington, DC

ContentsContinuing Medical Education Information .............................................................................. 2The Pathophysiology and Significance of Hyponatremia ...................................................... 3Case Study 1: Hyponatremia in SIADH ...................................................................................... 8Case Study 2: Hyponatremia in Heart Failure ........................................................................ 10Case Study 3: Hyponatremia in Neurocritical Care .............................................................. 12Case Study 4: Hyponatremia in Cirrhosis ................................................................................ 13Posttest ......................................................................................................................................... 15Evaluation Form ........................................................................................................................... 16

Joseph G. Verbalis, MD Editor

Paul J. Hauptman, MD

Stephan A. Mayer, MD, FCCM

Florence Wong, MBBS, MD, FRACP, FRCPC

Visit www.HyponatremiaCME.org for additional cases and activities

Page 2: Visit Clinical Perspectives in ... · Clinical Perspectives in Hyponatremia n March 2012 3 The PaThoPhysiology and significance of hyPonaTremia JosePh g. Verbalis, md Clinical Perspectives

2 Clinical Perspectives in Hyponatremia n March 2012

Clinical Perspectives in HyponatremiaContinuing Medical Education Information

Target AudienceThis activity has been designed to meet the educational needs of cardiologists, endocrinologists, critical care specialists, hospitalists, nephrologists, hepatologists, and hospital-based internists who manage patients with hyponatremia.

Educational ObjectivesAfter completing this activity, participants should be better able to:• Evaluate available therapeutic agents and approaches used in the management

of hyponatremia, including clinical benefits and potential adverse events• Develop a treatment plan for patients with hyponatremia

Instructions for ParticipationTo receive a CME certificate of participation, participants should:• Read the entire publication, including the Continuing Medical Education Information.• Complete the posttest and evaluation form on page 16 and mail or fax the evaluation form

with answer key to: Paradigm Medical Communications, LLC, 523 Route 303, Orangeburg, NY 10962; Fax: 845-398-5108.

A certificate of participation will be issued 4 to 6 weeks after receipt of a completed activityevaluation form and a completed posttest with a score of 70% or better.

Accreditation and Credit DesignationParadigm Medical Communications, LLC is accredited by the Accreditation Council for Continuing Medical Education to provide continuing medical education for physicians.Paradigm Medical Communications, LLC designates this enduring material for a maximum of 1.5 AMA PRA Category 1 Credit(s) TM. Physicians should claim only the credit commensurate with the extent of their participation in the activity.This activity is supported by an educational grant from Otsuka America Pharmaceutical, Inc.This educational activity is sponsored by Paradigm Medical Communications, LLC, Orangeburg, NY.Release Date: March 15, 2012Last Review Date: March 8 , 2012Expiration Date: March 14, 2013Estimated Time to Complete Activity: 1.5 hours

Contributing Faculty and DisclosuresIn accordance with ACCME requirements on disclosure, faculty and contributors are asked to disclose any relationships with commercial interests associated with the area of medicine featured in the activity.Joseph G. Verbalis, MDProfessor of Medicine and PhysiologyChief, Division of Endocrinology and MetabolismGeorgetown UniversityWashington, DCConsultant: Astellas Pharma US, Inc.; Cardiokine, Inc.; Otsuka America Pharmaceutical, Inc.

Paul J. Hauptman, MDProfessor of MedicineDivision of CardiologyDepartment of Internal MedicineSt. Louis University School of MedicineSt. Louis, MOConsultant/Speaker: Otsuka America Pharmaceutical, Inc.

Stephan A. Mayer, MD, FCCMProfessor of Neurology and NeurosurgeryColumbia University College of Physicians and SurgeonsDirector, Neurocritical CareNew York Presbyterian HospitalColumbia University Medical CenterNew York, NYConsultant: SanofiSpeaker: Astellas Pharma US Inc.

Florence Wong, MBBS, MD, FRACP, FRCPCProfessorDivision of GastroenterologyDepartment of MedicineUniversity of TorontoToronto, Ontario, CanadaNo financial relationships to disclose.

Independent peer reviewer: No financial relationships to disclose.

No Paradigm Medical Communications staff member who was in a position to control or influencethe content of this educational activity has a conflict of interest.

This newsletter is published by Paradigm Medical Communications, LLC, Orangeburg, NY.© 2012 Paradigm Medical Communications, LLC, except where noted. This newsletter may not be reproduced in whole or in part without the express written permission of Paradigm Medical Communications, LLC. This CME program represents the views and opin-ions of the individual faculty and does not constitute the opinion or endorsement of, or promotion by, Paradigm Medical Communications, LLC.Reasonable efforts have been taken to present educational subject matter in a balanced, unbiased fashion and in compliance with regulatory requirements. However, each activity participant must always use his or her own personal and professional judgment when considering further application of this information, particularly as it may relate to patient diagnostic or treatment decisions, including without limitation, FDA-approved uses and any off-label uses.

Publication Staff

PublisherParadigm Medical Communications, LLC523 Route 303Orangeburg, NY [email protected]

Senior Program DirectorAlison McMorrowNo financial relationships to disclose

EditorNancy LucasNo financial relationships to disclose

Contributing EditorDeborah KaplanNo financial relationships to disclose

Art DirectorWilliam ShannonNo financial relationships to disclose

Page 3: Visit Clinical Perspectives in ... · Clinical Perspectives in Hyponatremia n March 2012 3 The PaThoPhysiology and significance of hyPonaTremia JosePh g. Verbalis, md Clinical Perspectives

Clinical Perspectives in Hyponatremia n March 2012 3

The PaThoPhysiology and significance of hyPonaTremia JosePh g. Verbalis, md

Clinical Perspectives in Hyponatremia

Hyponatremia is the most com-mon electrolyte abnormality en-countered in clinical practice. The

disorder is defined as a serum sodium concentration ([Na+]) of <135 mEq/L. Epidemiologic studies reveal that hypo-natremia is a common problem in hos-pitalized patients, whether it is present at admission or hospital-acquired. The dis-order increases the risk of admission to the intensive care unit (ICU) and is asso-ciated with significantly increased mor-bidity and mortality and longer hospital length of stay.

Hyponatremia also appears to be a marker for more severe underlying dis-ease with a poorer prognosis. Heart fail-ure (HF), cirrhosis, and neurologic disease are among the serious clinical conditions known to be associated with hyponatre-mia. Chronic hyponatremia presents its own set of challenges because even in mild diseases that are often asymptom-atic, hyponatremic patients are at height-ened risk for gait disturbances, attention deficits, falls, and fractures.

The relationship between hyponatremia and water homeostasisIn the normal physiologic state, total body water accounts for about 60% of body weight depending on sex, body mass in-dex, and age. The level of water in the body is controlled by a single hormone, arginine vasopressin (AVP), also called antidiuretic hormone (ADH). To under-stand water homeostasis, it is important to understand AVP, how it is released, and its pathophysiologic influence.

AVP is made in the hypothalamus and transported down the pituitary stalk to the posterior pituitary, where it is re-leased. A variety of excitatory and in-hibitory factors regulate AVP secretion. Once released into the bloodstream, AVP affects various parts of the body by interacting with receptors in different ar-eas. The major site of action of the AVP V2 receptors, which affect water homeo-stasis by regulating free water absorption, is the renal collecting duct.

Under normal circumstances with-out AVP present, the collecting duct is

relatively impermeable to water. Any wa-ter in the nephron that makes it from the glomerulus and proximal tubule into the collecting duct is not reabsorbed, but is excreted via a process known as diuresis.

However, in response to AVP bind-ing to the V2 receptor, a signal transduc-tion cascade is activated with generation of cyclic AMP in the collecting duct prin-cipal cells, which results in insertion of aquaporin-2 water channels into the api-cal membrane of these cells (Figure 1). This allows a passive reabsorption of wa-ter along osmotic gradients into the cells, and then out the basolateral side of cells through other water channels that are constitutively expressed. The net result is reabsorption of water into the circulation, which is called antidiuresis.

Stimuli to AVP secretion related to fluid homeostasis include hyperosmolali-ty, hypotension, angiotensin, and hypovo-lemia. In response to these stimuli, AVP is secreted to conserve water and prevent further dehydration and volume depletion to maintain normal body water homeo-stasis. Stimuli independent of fluid ho-meostasis include nausea, hypoxia, hyper-carbia, hypoglycemia, stress (cytokines), and physical activity. These nonosmotic stimuli result in AVP secretion when it is not needed for body fluid homeostasis.

Pathophysiology and role of AVPToo little AVP secretion is associated with diabetes insipidus (DI)—there is exces-sive urination of water, because water cannot be reabsorbed by the kidney. Al-

to their serum osmolality. In normal indi-viduals, AVP levels should be suppressed to less than 0.5 pg/mL, which is the detec-tion limit for AVP, when serum osmolality falls below approximately 280 mOsm/kg H2O. When it is not suppressed, an os-motically inappropriate AVP secretion oc-curs, which in turn causes water retention and a dilutional hyponatremia.

A study by Robertson demonstrated that 95% of dilutional hyponatremia is caused by inappropriate, or nonosmotic, secretion of AVP.1 Studies in both HF and cirrhosis showed a similar pattern of osmotically inappropriate AVP secretion. The role of AVP in water retention was explored in an early study of 37 hypona-tremic HF patients; AVP was detected in 30 by immunoassay.2 Hyponatremia and hypo-osmolality were more severe, as was renal impairment, in the 30 patients with detectable AVP than in the 7 patients with undetectable AVP. Elevated AVP levels in HF did not appear to be consistently related to the use of diuretics.2 Just as in SIADH, plasma AVP levels are elevated despite normal low plasma osmolality.

The kidney is exquisitely sensitive to very low levels of AVP. As plasma osmo-lality increases there are linear increases in plasma AVP and urine osmolality, which can increase from 100 to more than 1000 mOsm/kg H2O. However, urine volume decreases inversely as AVP levels increase. With complete absence of AVP, as in patients with DI, the kidney has the capacity to excrete up to 1000 mL/h. But an AVP level of only 0.5 pg/

AP2

PKA

H2OH2O

H2OH2O

H2OH2O

H2O

H2O

Urinary space

H2O

Basolateral space

= V2 receptor antagonist

AVP

AVP

AVPG protein

Adenyl cyclase

cAMP

ATP

SV2 receptor

S

AP2, aquaporin-2; PKA, protein kinase A; cAMP, cyclic adenosine monophosphate;ATP, adenosine triphosphate; AVP, arginine vasopressin.

0

5

10

15

20

25

30

Pre

vale

nce

%M

orta

lity

%

Acute hospital care

Ambulatory hospital care

Community care

< 116 <135 >145 >165

V2 receptor

0.49

0

10

20

30

40

50

60Normonatremic

Hyponatremic

Heart failure13 Pulmonarytuberculosis14

Child diarrhea15 Myocardialinfarction16

10

35

4

50

2

10 8

20

0.17 0.03

28.2

21

7.2

1.430.53 0.72

0.06 0.01 0.01

Add to thenumerator

Na+E + K+

E

Total body waterSerum [Na+] =

Subtract from the denominator

Figure 1. Collecting Principal Duct Cellternately, nonosmotic AVP secretion, which causes wa-ter retention when it is not physiologically required, is associated with the syn-drome of inappropriate antidiuretic hormone se-cretion (SIADH), HF, and cirrhosis.

SIADH is the quint-essential example of a dilutional hyponatremia. Patients with SIADH have inappropriately elevated se-rum AVP levels in relation

Page 4: Visit Clinical Perspectives in ... · Clinical Perspectives in Hyponatremia n March 2012 3 The PaThoPhysiology and significance of hyPonaTremia JosePh g. Verbalis, md Clinical Perspectives

4 Clinical Perspectives in Hyponatremia n March 2012

Clinical Perspectives in Hyponatremia

mL—near the detection limit—halves the urinary excretion rate to 500 mL/h. With a further increase to 1 pg/mL, urinary ex-cretion decreases to 250 mL/h. Therefore, AVP levels of 1 to 3 pg/mL often report-ed in patients with SIADH are capable of substantial water retention by the kidney.

Incidence and prevalenceMany studies support the statement that hyponatremia is the most com-mon electrolyte disorder seen in clinical practice. In a 2003 study in Singapore, the prevalence of hyponatremia ([Na+] <135 mEq/L) was 28% in hospitalized patients, 21% in hospital clinics, and 7% in outpatient community care clinics.3

Many disorders can cause nonos-motic secretion of AVP and therefore potentially result in water retention and hyponatremia, including pulmonary dis-orders, tumors that sometimes synthe-size AVP, central nervous system (CNS) disorders that can dysregulate pituitary function, drugs, and a range of diverse conditions from the postoperative state to prolonged exercise to severe nausea.

At a serum [Na+] <136 mEq/L, the incidence of hyponatremia in HF was 21% in the Acute and Chronic Thera-peutic Impact of Vasopressin Antago-nist study of 319 hospitalized patients4 and 28% in 254 HF patients enrolled in a related study.5 The Acute Decompen-sated Heart Failure National Registry of 62,018 HF patients showed a 5% inci-dence of hyponatremia on entry with a lower serum [Na+] of <130 mEq/L.6

Hyponatremia also occurs in cirrho-sis, another edema-forming disorder. One

study showed that almost 50% of pa-tients with liver cirrhosis and ascites had hyponatremia.7 Most were in the relatively mild range of serum [Na+] from 131-135 mEq/L, but approximately 20% had a se-rum [Na+] <130 mEq/L.

The elderly are particularly suscepti-ble to hyponatremia. The overall incidence has been estimated at 7% in the geriatric population,8 but increases to 18% to 22% in chronic care facilities.9 The incidence of hyponatremic episodes can reach levels as high as 53% in nursing homes.10 Impor-tantly, the mortality rate is doubled (16% vs 8%) for hyponatremic patients over age 65 compared with those without hypona-tremia on admission to the hospital.11

In several studies of elderly patients with SIADH, hyponatremia was idiopath-ic. A 1997 study demonstrated that about 60% of patients over age 65 with SIADH had no discoverable cause of hyponatre-mia—no offending drugs, no pneumo-nia, no carcinoma, and none of the usual causes of SIADH.12 Without an underly-ing cause, these patients will be suscepti-ble to hyponatremia for a prolonged time because there is no possible treatment for an underlying disease to eliminate the in-appropriate AVP secretion.

In hospitalized patients, hyponatre-mia is a marker for higher mortality and adverse outcomes compared with those who are not hyponatremic whether the disease is HF,13 pulmonary tuberculo-sis,14 childhood diarrhea,15 or myocardial infarction16 (Figure 2). In one study, HF patients who were hyponatremic (de-fined as [Na+] <130 mEq/L) had much worse survival rates over a 3-year period

than those who presented initially with a serum [Na+] >130 mEq/L.17

Hyponatremia has also been found to be an independent predictor of mor-tality in liver failure.18 For example, in normonatremic patients awaiting liver transplantation with a MELD score of 25 to 29, mortality rates were 25% but increased to 66% for patients with hy-ponatremia. For patients with MELD scores above 30, mortality rates were 50% in normonatremic patients, but 100% in hyponatremic patients.18

Acute versus chronic hyponatremiaIt is crucial to differentiate the symp-toms of acute hyponatremia from those of chronic hyponatremia. Acute hypona-tremia can cause seizures, coma, respira-tory arrest and death. These symptoms reflect cerebral edema, which can poten-tially lead to herniation of the brain into the brain stem with consequent respira-tory arrest and death. The cerebral ede-ma may be accompanied by neurogenic pulmonary edema, and the resulting hypoxia can worsen the severity of the brain swelling and therefore the rapidity of death from respiratory arrest.

In chronic hyponatremia, symptoms are much less severe. In a study of 14 patients with acute hyponatremia of less than 12 hours’ duration and 52 patients with chronic hyponatremia of more than 3 days’ duration, stupor or coma occurred in only 6% with chronic hyponatremia compared with 100% with acute hypona-tremia; the mortality rate was 50% in the acute state, 6% in the chronic condition.19

Brain volume regulationThe reason for the profound difference between the symptoms of acute and chronic hyponatremia is the process of brain volume regulation. As extracellular [Na+] decreases, whether because of a loss of sodium or a gain of water, there is an obligate movement of water into the brain along osmotic gradients, which causes cerebral edema. When this is great-er than approximately 8%, the capacity of the skull to accommodate brain expan-sion is exceeded, leading to herniation and death from respiratory arrest.

Fortunately, not all hyponatremic patients die from cerebral edema be-cause the brain undergoes brain volume regulation, a process in which electro-

AP2

PKA

H2OH2O

H2OH2O

H2OH2O

H2O

H2O

Urinary space

H2O

Basolateral space

= V2 receptor antagonist

AVP

AVP

AVPG protein

Adenyl cyclase

cAMP

ATP

SV2 receptor

S

AP2, aquaporin-2; PKA, protein kinase A; cAMP, cyclic adenosine monophosphate;ATP, adenosine triphosphate; AVP, arginine vasopressin.

0

5

10

15

20

25

30

Pre

vale

nce

%M

orta

lity

%

Acute hospital care

Ambulatory hospital care

Community care

< 116 <135 >145 >165

V2 receptor

0.49

0

10

20

30

40

50

60Normonatremic

Hyponatremic

Heart failure13 Pulmonarytuberculosis14

Child diarrhea15 Myocardialinfarction16

10

35

4

50

2

10 8

20

0.17 0.03

28.2

21

7.2

1.430.53 0.72

0.06 0.01 0.01

Add to thenumerator

Na+E + K+

E

Total body waterSerum [Na+] =

Subtract from the denominator

Figure 2. Hyponatremia Is Associated with Higher Mortality13-16

Page 5: Visit Clinical Perspectives in ... · Clinical Perspectives in Hyponatremia n March 2012 3 The PaThoPhysiology and significance of hyPonaTremia JosePh g. Verbalis, md Clinical Perspectives

Clinical Perspectives in Hyponatremia n March 2012 5

lytes and organic osmolytes are excreted from the brain as it swells.20 As the solute is lost, the brain eliminates excess water, cerebral edema resolves, and osmolality is balanced across the extracellular fluid spaces inside and outside the brain.

Although these patients are no lon-ger at risk for life-threatening complica-tions, they are still subject to lesser but still impairing neurologic symptoms of more chronic hyponatremia including headache, nausea and vomiting, mental slowing, unstable gait and falls, confu-sion and delirium, and disorientation.

Often it is not evident when a pa-tient presents to the hospital whether the hyponatremia is acute or chronic be-cause the duration of the patient’s condi-tion is unknown. Therefore, the degree of symptomatology can serve as a sur-rogate for the duration of the hyponatre-mia because it reflects whether the brain has undergone volume regulation or not.

Brain volume regulation is clearly important because it allows hyponatremic patients to survive by eliminating brain edema. However, many remain symp-tomatic despite brain volume regulation. In a report of 223 patients with hypona-tremia induced by thiazide, hyponatremia was accompanied by a high incidence of symptoms.21 While dizziness can poten-tially be attributed to diuretic-induced hy-povolemia, symptoms such as confusion, obtundation, and seizures are more con-sistent with hyponatremic symptomatol-ogy. Because thiazide-induced hyponatre-mia can be readily corrected by stopping the drug and/or administering sodium, it represents an ideal condition in which to assess improvement in symptomatol-ogy with normalization of [Na+]. In this study, all symptoms resolved fol-lowing correction of the hyponatremia, indicating that hyponatremia was indeed responsible for them.

However, the “cost” of this adap-tive process is a brain that is chronically depleted of many organic solutes. In our lab, hyponatremic animals experienced a 30% depletion of glutamate, the major excitatory amino acid neurotransmitter in the brain that is responsible for all motor movements.22 Thus, chronic hyponatre-mia may be associated with neurocogni-tive and motor defects because the chron-ically hyponatremic brain is not normal; it has accommodated to an external threat

and survived, but in a compensated state. This is a classic example of “allostasis,” and the allostatic burden of the compen-sation is a solute-depleted brain.

One possible effect of the loss of brain amino acids including glutamate in patients with hyponatremia was demon-strated by a seminal study from Belgium.23 A group of 16 patients judged to have asymptomatic hyponatremia by normal neurologic exam underwent a battery of neurocognitive tests, which demonstrated delayed reaction times to defined stimuli. In addition, the patients walked a tandem gait, heel-to-toe, eyes open, on a sensorized mat that measured the center of gravity on the ball of their feet. The researchers then were able to see deviations of their center of gravity off the tandem line by measur-

ing the total traveled way (TTW). Patients with chronic asymptomatic hyponatremia (mean [Na+] 128 mEq/L) had a TTW of 1336 mm, which was significantly differ-ent from age-matched normal controls ([Na+] 140 mEq/L) whose TTW was 1035 mm. When the investigators corrected the hyponatremia in those 16 patients with a variety of different treatments, including AVP receptor antagonists, their TTW was equivalent to the normal controls. Thus, some hyponatremic patients have abnor-mal gait instability that corrects with cor-rection of their hyponatremia.

The functional significance of this gait instability is illustrated in Ren-neboog’s accompanying study of 122 patients with a variety of levels of hypo-natremia, all judged to be asymptomatic at the time of their emergency depart-ment (ED) visit.23 Researchers compared these patients with 244 normonatremic controls matched for age, sex, and under-lying disease also presenting to the ED during the same time period. The inci-dence of falls was 21.3% in hyponatremic patients and 5.3% in normonatremic controls. The adjusted odds ratio (OR) for presenting to the ED with a fall was 67-fold higher in hyponatremic patients

than in normonatremic patients.23

Kengne and colleagues explored the potential consequences of falls in 513 patients who presented with a fall and a resulting fracture and in age-matched con-trol patients who did not have a fall and fracture.24 The incidence of hyponatremia was significantly higher in patients present-ing with falls and fractures than in controls (13.6% vs 4%). A similar study from Lenox Hill Hospital in New York showed analo-gous results.25 In a study of 1408 female patients with chronic kidney disease, a se-rum [Na+] <135 mEq/L. was also clearly shown to be independently associated with an increased risk of fractures.26

In addition to falls and fractures, osteoporosis has been associated with chronic hyponatremia. In our laboratory, we found that hyponatremia induced dramatic bone loss as a result of a 5-fold increase in osteoclasts in trabecular bone in rats who were hyponatremic for 3 months.27 Data analysis from the Third National Health and Nutrition Examina-tion Survey found a significant increased OR of 2.87 for the occurrence of osteo-porosis in patients with hyponatremia even though their mean serum [Na+] was only 133 mEq/L.27 Thus, even very mild levels of hyponatremia appear to be as-sociated with worsened osteoporosis.

One of the potential complications of correcting hyponatremia is the os-motic demyelination syndrome (ODS), also known as pontine and extrapon-tine myelinolysis. Symptoms include tremor, incontinence, and hyperreflexia or pathologic reflexia, eventually result-ing in quadriparesis with dysarthria, dys-phagia, cranial nerve palsies, and mutism or locked-in syndrome. Symptoms oc-cur because of demyelination of motor axons in the pons. Clinically, the occur-rence of central pontine myelinolysis in a setting of severe metabolic derangement, particularly of serum sodium, has been noted.28 We know from animal stud-ies that hyponatremic animals are much more susceptible to brain dehydration with acute correction of the hyponatre-mia. The brains of hyponatremic rats shrink even more as the plasma [Na+] is raised, because they have lost a substan-tial part of their osmotic buffering capac-ity during the process of brain volume regulation via solute losses.29

In animal studies, Adler and col-

In hospitalized patients,hyponatremia is a marker for higher mortality and adverse outcomes.

Page 6: Visit Clinical Perspectives in ... · Clinical Perspectives in Hyponatremia n March 2012 3 The PaThoPhysiology and significance of hyPonaTremia JosePh g. Verbalis, md Clinical Perspectives

6 Clinical Perspectives in Hyponatremia n March 2012

Clinical Perspectives in Hyponatremia

leagues investigated whether such shrink-age might disrupt the blood-brain barrier (BBB).30 BBB intactness was evaluated using MRI following IV gadolinium con-trast administration. The group noted that hypertonic saline infusion rapidly increased the plasma [Na+] and caused BBB disruption more frequently in chronic than in acutely hyponatremic rats. Similar increases in plasma [Na+] did not disrupt the BBB in normonatremic rats. The disruption appeared to be due to altered plasma osmolality, since infusion of hypertonic mannitol, which raised plasma osmolality without changing the plasma [Na+], also disrupted the BBB in hyponatremic but not normonatremic rats. Moreover, the osmotic threshold for BBB disruption was lowest in chronic hy-ponatremia, intermediate in acute hypo-natremia, and highest in normonatremia.

The greater susceptibility to osmotic BBB disruption in chronic hyponatremia suggests that BBB disruption may play a significant role in causing the demyelin-ation sometimes found following too rapid correction of hyponatremia.

Current therapies of hyponatremia and medical need for new therapiesUntil recently, the treatments available for hyponatremia included isotonic and hyper-tonic saline for short-term use, and fluid restriction, demeclocycline, furosemide, salt tablets, and rarely mineralocorticoids and urea for long-term use. All of these therapies work in specific circumstances, but none are ideal and all have limitations for a variety of different reasons, including variable efficacy, slow responses, intolerable side effects such as thirst, and dangerous toxicities such as ODS. In the last several years a new class of agents, AVP receptor antagonists, or vaptans, have become avail-able for treating hyponatremia: conivaptan for short-term use, and tolvaptan for both short- and long-term use. Of note, all treat-ments other than demeclocycline and the vaptans fail to attack the mechanistic cause of dilutional hyponatremia, which is inap-propriately elevated plasma AVP levels.

Fluid restriction, long the mainstay of treating chronic hyponatremia, is ar-guably the least expensive and potentially the safest treatment. But its major draw-back, reported as early as in the initial de-scription of SIADH in 1957, is that fluid restriction usually cannot be sustained

AP2

PKA

H2OH2O

H2OH2O

H2OH2O

H2O

H2O

Urinary space

H2O

Basolateral space

= V2 receptor antagonist

AVP

AVP

AVPG protein

Adenyl cyclase

cAMP

ATP

SV2 receptor

S

AP2, aquaporin-2; PKA, protein kinase A; cAMP, cyclic adenosine monophosphate;ATP, adenosine triphosphate; AVP, arginine vasopressin.

0

5

10

15

20

25

30

Pre

vale

nce

%M

orta

lity

%

Acute hospital care

Ambulatory hospital care

Community care

< 116 <135 >145 >165

V2 receptor

0.49

0

10

20

30

40

50

60Normonatremic

Hyponatremic

Heart failure13 Pulmonarytuberculosis14

Child diarrhea15 Myocardialinfarction16

10

35

4

50

2

10 8

20

0.17 0.03

28.2

21

7.2

1.430.53 0.72

0.06 0.01 0.01

Add to thenumerator

Na+E + K+

E

Total body waterSerum [Na+] =

Subtract from the denominator

Figure 3. Correcting Hyponatremia

because it produces intolerable dehydra-tion and thirst.31 Secondly, fluid restric-tion works very slowly—generally 1 to 2 mEq/L per day even under a severe fluid restriction of <500 mL/d. Conse-quently, this method can take many days to correct a low serum [Na+].

While demeclocyline targets the patho-physiology of hyponatremia by causing a postreceptor defect in the collecting duct cell, thereby impairing the concentration process, it is not FDA-approved for treat-ment of hyponatremia.

The recently approved vaptans are called aquaretic agents. Conivaptan is a combined antagonist of the V1a and V2 receptors, and tolvaptan is a selective an-tagonist of the V2 receptor. Another V2 an-tagonist, lixivaptan, is still in clinical trials.

The rationale for aquaretic therapies Aquaretic therapies can be best appreci-ated by an analysis of the determinants of the serum [Na+], which is the ratio of exchangeable sodium and potassium di-vided by total body water. Given this rela-tionship, there are only two basic ways to raise the serum [Na+] (Figure 3). One is in-creasing the numerator by infusing either sodium or potassium, which is efficacious for patients who are solute-depleted, often due to diuretic use. However, in numerous studies, up to two thirds of patients with hyponatremia are not solute-depleted but rather have water expansion as the cause of their hyponatremia. For those patients, the second strategy is the most appropri-ate: decreasing the denominator by re-moving excess total body water.

That is the objective of fluid re-striction that decreases body water via insensible losses of water, but very inef-

ficiently and very slowly. A better ther-apy would be blocking the process of urinary concentration, which originates from AVP binding to the V2 receptor in the kidney and would prevent all the downstream intracellular consequences of V2 receptor activation including the insertion of the aquaporin water chan-nels into the apical membrane of the collecting duct cells. That is, in fact, the mechanism of action of the vap-tans, which stimulate increased water excretion from the kidney. Importantly, blocking the V2 receptor has no effect on sodium or potassium excretion; it simply causes increased excretion of wa-ter without increased solute. Therefore, vaptans are electrolyte-sparing, and it is more appropriate to call their effect “aquaresis” or increased urinary water excretion rather than diuresis, which classically has been defined as increased urinary water and solute (particularly so-dium and potassium) excretion.

Patients with severe symptoms (eg, vomiting, seizures, obtundation, respira-tory distress, coma) typically associated with acute hyponatremia are at acute risk of death and morbidity from other ad-verse neurologic outcomes, necessitating prompt treatment.32 Although the vap-tans are able to raise serum [Na+] fairly quickly—within 8 hours—hypertonic saline is even quicker and can increase serum [Na+] within minutes, and is there-fore the treatment of choice. Once the serum [Na+] corrects to the desired level and patients are out of trouble, fluid re-striction or, if necessary, a vaptan can then be used to further increase serum [Na+] to a normal level, or to maintain it.

More moderate symptoms (eg, nau-sea, confusion, disorientation, altered mental status)—that are not life-threat-ening but can impair a patient’s ability to function—can be treated with a vaptan, followed if necessary by fluid restriction or sometimes by chronic vaptan use. Vaptans are suggested in these patients because it will generally take too long for fluid restriction to correct the serum [Na+] to normal ranges. Safely improv-ing serum [Na+] to increased or normal ranges within 1 to 2 days is the most ap-propriate therapy to relieve moderate neurologic symptoms.

Finally, patients with minimal symp-toms (eg, headache, irritability, inability to

Page 7: Visit Clinical Perspectives in ... · Clinical Perspectives in Hyponatremia n March 2012 3 The PaThoPhysiology and significance of hyPonaTremia JosePh g. Verbalis, md Clinical Perspectives

Clinical Perspectives in Hyponatremia n March 2012 7

concentrate, altered mood, depression) or no symptoms at all, are most appropriately treated initially with fluid restriction. How-ever, one should consider use of a vaptan even with mild or no symptoms under se-lect circumstances, including: n Inability to tolerate fluid restriction or a failure of fluid restriction; n Patients with a high fracture risk, osteoporosis, and any degree of gait instability or history of falls with hyponatremia; n Serum [Na+] <125 mEq/L, because these patients are at higher risk for developing more symptomatic levels of hyponatremia; n Correcting serum [Na+] to safer levels for surgery, anesthesia, and other procedures; n Prevention of worsening hypona- tremia with forced fluid administra- tion, typically for patients on par- enteral nutrition (which increases their volume load); n To put patients in a safer [Na+] range for ICU or hospital discharge; and n As a therapeutic trial in patients who have neurologic symptoms when it is not clear whether the cause is the hyponatremia or not.

Patients should be in a hospital setting for initiation or re-initiation of therapy both to evaluate the therapeutic response and because too rapid correc-tion of hyponatremia can cause ODS (ie, a serum [Na+] increase >12 mEq/L in 24 h, or >8 mEq/L in patients with risk fac-tors for ODS). The serum [Na+] must be monitored at regular intervals, at least ev-ery 8 hours but more frequently in higher risk patients, to make sure correction stays within desired and safe limits. If it is proceeding more rapidly than planned, additional water should be administered, either orally or IV as 5% dextrose. For the same reason, patients started on vap-tans should not be fluid restricted initially, since the increased thirst that accompa-nies increases in serum [Na+] will blunt further correction of the serum [Na+].

The need to use tolvaptan, or other therapies, for long-term therapy depends on whether the cause of the hyponatre-mia is chronic. Some types of hypona-tremia (eg, postoperative hyponatremia, exercise-associated hyponatremia, pneu-monia-associated hyponatremia) usually

resolve within days; therefore the likeli-hood of chronic hyponatremia is very low and there should be no indication or need for long-term treatment with vaptans.33 Alternately, hyponatremia caused by tu-mors, CNS disorders, HF, and cirrhosis is more typically chronic, requiring some form of long-term therapy.

The general guidelines for fluid re-striction are to restrict all intake consumed by drinking (Table 1). Sodium restriction is not indicated for typical patients with SIADH, because they generally do not have excess sodium or edema. In fact, so-dium should be liberalized unless contra-indicated. Obviously, this does not apply to hyponatremic patients with HF or cir-rhosis. If a patient fails fluid restriction, it is appropriate to consider treatment with a vaptan. This can usually be determined by the initial response to fluid restriction in the first 1 to 2 days (Table 1), rather than waiting 5 to 7 days before concluding that fluid restriction is a failure, which will un-necessarily prolong hospitalization.

References1. Robertson GL, Aycinena P, Zerbe RL. Neurogenic disor-ders of osmoregulation. Am J Med. 1982;72(2):339-53.2. Szatalowicz VL, Arnold PE, Chaimovitz C, Bichet D, Berl T, Schrier RW. Radioimmunoassay of plasma arginine va-sopressin in hyponatremic patients with congestive heart failure. N Engl J Med. 1981;305(5):263-6.3. Hawkins RC. Age and gender as risk factors for hypo-natremia and hypernatremia. Clin Chim Acta. 2003;337(1-2):169-72. 4. Gheorghiade M, Gattis WA, O’Connor CM, Adams KF Jr, et al. Effects of tolvaptan, a vasopressin antagonist, in patients hospitalized with worsening heart failure: a randomized controlled trial. JAMA. 2004;291(16):1963-71.5. Gheorghiade M, Niazi I, Ouyang J, Czerwiec F, et al. Va-sopressin V2-receptor blockade with tolvaptan in patients

with chronic heart failure: results from a double-blind, randomized trial. Circulation. 2003;107:2690-6.6. ADHERE Registry. 3rd Quarter 2003 National Benchmark Report. Sunnyvale CA: Scios, Inc. 2004.7. Angeli P, Wong F, Watson H, Gines P, and the CAPPS In-vestigators.Hyponatremia in cirrhosis: results of a patient population survey. Hepatology. 2006;44:1535-42.8. Caird FI, Andrews GR, Kennedy RD. Effect of posture on blood pressure in the elderly. Br Heart J. 1973;35:527-30. 9. Kleinfeld M, Casimir A, Borra S. Hyponatremia as ob-served in a chronic disease facility. J Am Geriatr Soc. 1979;27(4):156-61.10. Miller M, Morley JE, Rubenstein LZ. Hyponatre-mia in a nursing home population. J Am Geriatr Soc. 1995;43(12):1410-13. 11. Terzian C, Frye EB, Piotrowski ZH. Admission hypona-tremia in the elderly: factors influencing prognosis. J Gen Intern Med. 1994;9(2):89-91.12. Hirshberg B, Ben-Yehuda A. The syndrome of inappro-priate antidiuretic hormone secretion in the elderly. Am J Med. 1997;103:270-3.13. Flear CT, Singh CM.The sick cell concept and hypo-natremia in congestive heart failure and liver disease. Lancet. 1982;2(8289):101-2.14. Westwater JO, Stiven D, Garry RC. Serum sodium level in patients suffering from tuberculosis. Clin Sci. 1939;4:73-7.15. Samadi AR, Wahed MA, Islam MR, Ahmed SM. Con-sequences of hyponatraemia and hypernatraemia in children with acute diarrhoea in Bangladesh. Br Med J. 1983;286(6366):671-3.16. Flear CT, Hilton P. Hyponatraemia and sever-ity and outcome of myocardial infarction. Br Med J. 1979;1(6173):1242-6. 17. Lee WH, Packer Ml. Prognostic importance of serum sodium concentration and its modification by converting-enzyme inhibition in patients with severe chronic heart failure. Circulation. 1986;73(2):257-67.18. Ruf AE, Kremers WK, Chavez LL, Descalzi VI, et al. Addition of serum sodium into the MELD score predicts waiting list mortality better than MELD alone. Liver Trans-plantation. 2005;11(3):336-43. 19. Arieff AI, Llach F, Massry SG. Neurological manifes-tations and morbidity of hyponatremia: correlation with brain water and electrolytes. Medicine. 1976;55(2):121-9. 20. Gullans SR, Verbalis JG. Control of brain volume dur-ing hyperosmolar and hypoosmolar conditions. Annu Rev Med. 1993;44:289-301.21. Chow KM, Kwan BC, Szeto CC. Clinical studies of thiazide-induced hyponatremia. J Natl Med Assoc. 2004;96(10):1305-8. 22. Verbalis JG, Gullans SR. Hyponatremia causes large sustained reductions in brain content of multiple organic osmolytes in rats. Brain Res. 1991;567(2):274-82.23. Renneboog B, Musch W, Vandemergel X, Manto MU, Decaux G. Mild chronic hyponatremia is associated with falls, unsteadiness, and attention deficits. Am J Med. 2006;119(1):71.e1-71.e8.24. Kengne FG, Andres C, Sattar L, Melot C, et al. Mild hy-ponatremia and risk of fracture in the ambulatory elderly. QJM. 2008;101(7):583-8.25. Sandhu HS, Gilles E, DeVita MV, Panagopoulos G, Mi-chelis MF. Hyponatremia associated with large-bone frac-ture in elderly patients. Int Urol Nephrol. 2009;41(3):733-7.26. Kinsella S, Moran S, Sullivan MO, Molloy MG, Eustace JA. Hyponatremia independent of osteoporosis is as-sociated with fracture outcome. Clin J Am Soc Nephrol. 2010;5(2):275-80. 27. Verbalis JG, Barsony J, Sugimura Y, Tian Y, et al. Hyponatremia-induced osteoporosis. J Bone Miner Res. 2010;25(3):554-63. 28. Wright DG, Laureno R, Victor M. Pontine and extrapon-tine myelinolysis. Brain. 1979;10(2):361-85.29. Berl T. Treating hyponatremia: Damned if we do and damned if we don’t. Kidney Int. 1990;37(3):1006-18.30. Adler S, Verbalis JG, Williams D. Effect of rapid cor-rection of hyponatremia on the blood-brain barrier of rats. Brain Res. 1995;679:(1):135-43.

Table 1. Fluid Restriction

General guidelines

• Restrict all intake consumed by drinking, not just water

• Aim for a fluid restriction 500 mL/d below the 24-h urine output

• Do not restrict sodium unless indicated

Predictors of fluid restriction failure

• High urine osmolality (>500 mOsm/kg H2O)

• Sum of the urine [Na+] and [K+] is greater than the serum [Na+]

• 24-hour urine output <1,500 mL/d

• Increase in serum [Na+] <2 mEq/L in 24 h

Page 8: Visit Clinical Perspectives in ... · Clinical Perspectives in Hyponatremia n March 2012 3 The PaThoPhysiology and significance of hyPonaTremia JosePh g. Verbalis, md Clinical Perspectives

8 Clinical Perspectives in Hyponatremia n March 2012

Clinical Perspectives in Hyponatremia

case sTudy 1: hyPonaTremia in siadh JosePh g. Verbalis, md

Mrs. T, a 40-year-old woman hos-pitalized 8 times from 2007 to 2009 for symptomatic hypona-

tremia with serum [Na+] as low as 117 mEq/L and corresponding neurologic symptoms, presents again with severe hyponatremia. Despite treatment with demeclocycline, 1200 mg/d, and fluid restriction, the patient’s serum [Na+] has remained at 129 mEq/L. She was not on any other medications.

Initial evaluationMrs. T had a serum [Na+] of 129 mEq/L on demeclocycline and fluid restriction, a plasma osmolality of 270 mOsm/kg H2O, a urine osmolality of 1159 mOsm/kg H2O, and a urine [Na+] of 247 mEq/L. Examination revealed that the patient was clinically euvolemic, without edema or ascites or orthostatic blood pressure changes. Thyroid function tests were normal. Normal cortisol results after a cosyntropin stimulation test indicated absence of adrenal insufficiency. Other routine laboratory tests (CBC, metabolic profile, liver functions) were all normal. A CT scan of the chest, abdomen, and pelvis was similarly normal. MRI of the brain was unrevealing, and all serologies for HIV and hepatitis were negative.

What is the most likely etiology? Given this dilemma of a relatively young female with severe hyponatremia consis-tent with SIADH, potential etiologies in-clude an activating mutation of the AVP V2 receptor gene; surreptitious diuretic use; ecstasy or other drug abuse; ectopic AVP production by a tumor; or idiopathic SIADH.

The nephrogenic syndrome of inap-propriate antidiuresis (NSIAD) is caused by an activating mutation of the AVP V2 receptor at the same site that also can cause DI via an inactivating mutation.1 However, this patient’s initial presenta-tion is not consistent with a genetic eti-ology because of the late age of onset of her hyponatremia. As for surreptitious

diuretic use, it would be very unusual for a patient on diuretics to present with a urine osmolality >1000 mOsm/kg H2O unless he or she was also very hypovo-lemic; but by clinical examination, this patient is euvolemic.

Drug-induced-hyponatremia is a com-mon problem, with many different drugs capable of causing hyponatremia by a num-ber of mechanisms, including volume de-pletion, stimulation of inappropriate AVP secretion, potentiation of AVP effects at the kidney, and changes in plasma osmolal-ity.2 Methamphetamines are associated with hyponatremia, but drug abuse is not the likely cause of this patient’s hyponatremia because its effect is relatively short-lived and hyponatremia is generally only present during the period of acute administration.

Body fluids are maintained within a remarkably narrow range of osmolal-ity (average basal level: 285-290 mOsm/kg H2O), primarily through the action of osmoreceptors in the hypothalamus that can detect small changes in plasma solute concentrations, and then transform this information into neural signals that either stimulate or inhibit both thirst and AVP release. Hypo-osmolar states, such as oc-cur in patients with SIADH, are primarily the result of dysregulated water excretion, which is usually caused by a defect in AVP osmoregulation. According to Robert- son’s 1982 study in 100 patients with SIADH, plasma AVP levels ranged wide-ly, but in most cases were still measurable when they should have been suppressed by the low plasma osmolality.3

In cases of SIADH, AVP is inad-equately suppressed despite low plasma osmolality, which causes inappropriate circulating levels of AVP relative to the plasma osmolality. The antidiuretic effect of AVP at the kidney then leads to wa-ter retention and subsequent dilution of body fluids (fall in plasma osmolality to <280 mOsm/kg H2O and plasma [Na+] to <135 mEq/L). Very high levels of AVP (>10 pg/mL) are generally indicative of ectopic AVP production. Although we

don’t have a measured AVP level in this case, the patient’s very high urine osmolal-ity indicates the AVP level is also high. A urine osmolality >1000 mOsm/kg H2O requires plasma AVP levels >5 pg/mL. Thus, the extremely high urine osmolal-ity in this patient is most consistent with SIADH from ectopic AVP production. The most common cause of ectopic AVP production is tumors.

Consequently, in this relatively young patient with a high urine osmolality indi-cating a high plasma AVP level, the most likely cause of SIADH is a tumor. Idio-pathic SIADH is far less likely because it is seen predominantly in much older patients (>65 years) and because patients rarely have elevated urine osmolalities to this degree.

What further evaluation should be undertaken?The patient has had extensive imag-ing of the chest, abdomen, pelvis, and brain with no abnormal results reported. Nonetheless, additional imaging and testing might be considered, including PET, lumbar puncture for cerebrospinal fluid cytology, MRI of the sinuses and nasopharynx to look for nasopharyngeal lesions, and a serum plasma carcinoem-bryonic antigen test to detect an occult gastrointestinal (GI) malignancy that may not have been picked up by initial imaging. Or, perhaps further imaging is not necessary because the etiology of the patient’s SIADH may not be a tumor.

Because brain tumors can cause SIADH, the recommended course for this patient was to revisit her MRI of the brain focusing on the nasopharynx. A second read of the imaging revealed a focal thickening of the superior nasal mucosa that was not mentioned on the initial read. An otolaryngology consul-tation advised resection of this lesion, which led to the pathologic diagnosis of esthesioneuroblastoma. After resection of the tumor, the patient’s hyponatremia completely resolved. At 1-year follow up,

31. Schwartz WB, Bennett W, Curelop S, Bartter FC. A syndrome of rneal soium loss and hyponatremia probably resulting from inappropriate secretion of antidiuretic syn-drome. Am J Med. 1957;23(4):529-42.

32. Verbalis JG, Goldsmith SR, Greenberg A, Schrier RW, Sterns RH. Hyponatremia treatment guidelines 2007: expert panel recommendations. Am J Med. 2007;120(11 Suppl 1):S1-21.

33. Verbalis JG. Managing hyponatremia in patients with syndrome of inappropriate antidiuretic hormone secretion. Endocrinol Nutr. 2010;57(Suppl 2):30-40.

Page 9: Visit Clinical Perspectives in ... · Clinical Perspectives in Hyponatremia n March 2012 3 The PaThoPhysiology and significance of hyPonaTremia JosePh g. Verbalis, md Clinical Perspectives

Clinical Perspectives in Hyponatremia n March 2012 9

the patient’s serum [Na+] remained nor-mal and she no longer required treatment with demeclocycline or fluid restriction.

Which tumors are associated with SIADH?SIADH is a leading cause of hyponatre-mia in patients with cancer.4,5 SIADH oc-curs with a wide range of cancers but the most common by far is small-cell lung cancer (11%–46%), followed by head and neck cancers (3%–5%). Cancers of the brain (both primary and metastatic), a variety of hematologic malignancies, intrathoracic but nonpulmonary tumors, and melanoma are rarely associated with SIADH. GI and genitourinary cancers are even more rarely associated with SIADH. The incidence of breast and ad-renal cancers with hyponatremia is very low, but these have been reported. Over-all, tumors account for about 30% of all cases of hyponatremia, and in general, 3% to 5% of the entire patient popula-tion with cancer has hyponatremia.6

Given the high frequency of SIADH in lung cancer, it was appropriate initially to do a chest CT scan in this patient, which has the highest yield in terms of imaging for tumors with hyponatremia. Next in importance for imaging is an MRI of the head because of the high frequency of head and neck cancers. The patient’s tu-mor, a cancer of the olfactory epithelium also called olfactory neuroblastoma, can present with loss of smell and visual dis-

turbances. It is one of the rarer types of head and neck cancers with fewer than 1000 cases reported. In all 9 previous reports published in association with hy-ponatremia secondary to SIADH, elec-trolyte abnormalities, namely hyponatre-mia, led to the discovery of the tumor.7

Many other factors also increase the risk for hyponatremia in cancer pa-tients. First, a variety of chemotherapies stimulate AVP secretion.8 In particular, immunomodulators such as interferon and interleukin-2 are associated with hy-ponatremia, as have been monoclonal antibodies.9 Second, the nausea and vom-iting that often occur with chemotherapy will independently elevate AVP levels and therefore can cause hyponatremia,10 particularly if hypotonic fluids are be-ing used to keep the patient hydrated.11 Third, hypotonic dehydration, another cause of hyponatremia, can occur as a result of the nephrotoxicity produced by many chemotherapeutic agents. Fourth, analgesic agents can stimulate AVP se-cretion, leading to water retention and SIADH,12 as can pain and physical stress, especially when coupled with free water administration.10

Future management options Therapeutic options for chronic hypo-natremia with recurrence of the esthe-sioneuroblastoma would include fluid restriction, demeclocycline, urea, hyper-tonic saline infusion, or an AVP receptor

antagonist (vaptan). Traditional treatments for hypona-

tremia such as fluid restriction, demeclo-cycline, urea, and hypertonic saline infu-sion have several significant limitations, including limited efficacy data, potential toxicities, and compliance difficulties.13

While each of these therapies can work in specific circumstances, none are ideal and all have significant limitations (Table 2). Only the vaptans and demeclocycline actually target the underlying pathophysi-ology of dilutional hyponatremia, namely inappropriately elevated AVP levels. Dem-eclocycline, however, exhibits significant nephrotoxicity and is not FDA approved for the treatment of hyponatremia.

In this patient, urea would be thera-peutic, but it is inconvenient and has poor palatability. Hypertonic saline and conivaptan are short-term therapies and could not be employed in this case. So long-term treatment with tolvaptan would be the only agent predicted to be success-ful in correcting this patient’s serum [Na+] and maintaining it within normal ranges over a long period.14

References1. Feldman BJ, Rosenthal SM, Vargas GA, Fenwick RG, Huang GA, et al. Nephrogenic syndrome of inappropriate antidiuresis. N Engl J Med. 2005;352:1884-92.2. Liamis G, Milionis H, Elisaf M. A review of drug-induced hyponatremia. Am J Kidney Dis. 2008;52:144-53.3. Robertson GL, Aycinena P, Zerbe RL. Neurogenic disor-ders of osmoregulation. Am J Med. 1982;72(2):339-53.4. Glover DJ, Glick JH. Metabolic oncologic emergencies. CA Cancer J Clin. 1987;37:302-20.5. Silverman P, Distelhorst CW. Metabolic emergencies in clinical oncology. Semin Oncol. 1989;16:504-15.6. Baylis PH. The syndrome of inappropriate antidiuretic hormone secretion. Int J Biochem Cell Biol. 2003;35:1495-9.7. Plasencia YL, Cortes MB, Aencibia DM, et al. Ethesio-neuroblastoma recurrence presenting as a syndrome of inappropriate antidiuretic hormone secretion. Head Neck. 2006;28:1142-6.8. Vanhees SL, Paridaens R, Vansteenkiste JF. Syndrome of inappropriate antidiuretic hormone associated with chemotherapy-induced tumor lysis in small-cell lung cancer: case report and literature review. Ann Oncol. 2000;11:1061-5.9. Berghmans T, Paesmans M, Body JJ. A prospective study on hyponatremia in medical cancer patients: epide-miology, etiology and differential diagnosis. Support Care Cancer. 1999;8:192-7.10. Decaux G, Soupart A. Treatment of symptomatic hypo-natremia. Am J Med. 2003;326:25-30.11. Bissett D, Cornford EJ, Sokal M. Hyponatremia follow-ing cisplatin chemotherapy. Acta Oncol. 1989;28:823.12. Langfeldt LA, Cooley ME. Syndrome of inappropriate antidiuretic hormone secretuion in malignancy: review and implications for nursing management. Clin J Oncol Nurs. 2003;7:425-30.13. Goldsmith SR. Current treatments and novel pharma-cologic treatments for hyponatremia in congestive heart failure. Am J Cardiol. 2005;95(suppl):14B-23B.14. Berl T, Quittnat-Pelletier F, Verbalis JG, Schrier RW, Bichet DG et al. Oral tolvaptan is safe and effective in chronic hyponatremia. J Am Soc Nephrol. 2010;21: 705-12.

Therapy Limitations

Isotonic saline Ineffective in dilutional hyponatremias; can’t be used in edema-forming disorders; no controlled safety database

Hypertonic saline No consensus re appropriate infusion rates; overcorrection can cause osmotic demyelination; can’t be used in edema-forming disorders; no controlled safety database

Fluid restriction Slow to correct over many days; poorly tolerated due to thirst; unsuccessful with high AVP levels and urine osmolalities

Demeclocycline Not FDA approved for hyponatremia; slow to correct; nephrotoxic in cirrhosis and CHF

Mineralocorticoid Only one report in elderly patients with SIADH; no safety database; can’t be used in edema-forming disorders

Urea No USP formulation; not FDA approved for hyponatremia; poor palatability

AVP receptor antagonist Conivaptan approved only for in-hospital use secondary to CYP3A4 inhibition; infusion-site reactions with IV use

Table 2. Treatments for Hyponatremia

Page 10: Visit Clinical Perspectives in ... · Clinical Perspectives in Hyponatremia n March 2012 3 The PaThoPhysiology and significance of hyPonaTremia JosePh g. Verbalis, md Clinical Perspectives

10 Clinical Perspectives in Hyponatremia n March 2012

Clinical Perspectives in Hyponatremia

Mrs. M, a 66-year-old woman with a history of severe mitral and tricus-pid regurgitation and right-sided

HF, S/P implantation of an implantable cardioverter defibrillator (ICD) presents with increased fatigue, decreased appe-tite, and the inability to put her shoes on comfortably because of lower extrem-ity edema. She has recently taken extra metolazone. Her husband reports that “She is not doing her crossword puzzles anymore.” Her weight has increased from 61 to 72 kg over the prior week. The patient is referred for inpatient man-agement.

Past medical historyThe patient had mitral and tricuspid valve repairs in 2005. Her course at that time was complicated by respiratory failure and intermittent heart block, for which she received a dual chamber permanent pacemaker; her left ventricular ejection fraction (LVEF) was 45% and the LV was mildly dilated. Her right ventricle was dilated and her right ventricular systolic pressure (RVSP) was 45 mm Hg.

Mrs. M has been chronically main-tained on an ACE inhibitor, a beta-blocker, digoxin, and furosemide. In the 18 months prior to this presentation, she had a worsening functional status and dyspnea with 3 hospitalizations for HF. Her serum creatinine varied from 1.1 to 1.5 mg/dL and was noted to be higher when her diuretic dose was increased. Her SBP ranged from 85 to 90 mm Hg; her electrocardiograms and pacemaker interrogations consistently revealed 30% atrioventricular (AV) pacing, but she had an underlying right bundle branch block and occasional paroxysms of atrial fibril-lation. An echocardiogram obtained dur-ing the second of the 3 hospitalizations revealed a LVEF of 20%, a dilated right ventricle, RVSP of 65 mm Hg, a moder-ate degree of mitral regurgitation without a significant gradient across her repaired mitral valve, and severe tricuspid regurgi-tation.

At that same time point, the patient was under consideration for an ICD. However, her cardiologist was uncertain about the advisability of placing an LV lead, partly for technical reasons and part-

ly because the underlying QRS morphol-ogy demonstrated a right, not left bundle branch block pattern. Addition of cardiac resynchonization therapy requires a wide QRS, though greatest clinical response appears to occur in the setting of a wide (>150 ms) left bundle branch block pat-tern.1 Further, implantations during a hospitalization for HF have a greater mortality rate than elective admissions.2

After due consideration, the patient con-sented to an ICD alone and she experi-enced no periprocedural complications.

Recent clinical course In the outpatient setting, the patient’s se-rum [Na+] values varied over time, from 130 to 134 mEq/L. She was compliant with medications and reasonably com-pliant with diet, but had a predilection for French fries and ketchup. Frequent modifications of diuretic dosing were attempted, including addition of meto-lazone, first on a once weekly basis and then every day 30 minutes prior to furo-semide.

Examination at presentationThe exam was notable for an elderly, frail female with a SBP of 92 mm Hg, heart rate of 96, marked jugular venous disten-sion and clear “v”-waves (from tricuspid regurgitation), presence of a Kussmaul’s sign, decreased breath sounds at the bases with egophony, a palpable liver, and 2+ bilateral lower extremity edema. She also had significant mitral and tricuspid regur-gitation murmurs and a right-sided third heart sound. She appeared to be slightly distracted and her gait was unsteady.

MedicationsOn admission the patient was on ramipril, 5 mg daily; metoprolol succinate, 25 mg once daily; digoxin, 0.125 mg every other day; isosorbide mononitrate, 30 mg daily; furosemide, 100 mg daily; and metola-zone, 2.5 mg once a week. She also took lorazepam as needed.

Initial laboratory findingsMrs. M had a serum [Na+] of 124 mEq/L, a serum [K+] of 3.6 mEq/L, a BUN of 44 mg/dL, a bilirubin of 1.9 mg/dL, and her alkaline phosplatase level was normal.

What is the significance of the patient’s hyponatremia?Hyponatremia is much more than an iso-lated lab abnormality; it is a potent prog-nostic marker.3 It may cause nonspecific signs and symptoms including confusion and gait disturbance. Multiple studies de-rived from randomized clinical trials, reg-istries, and observational databases have demonstrated that baseline serum [Na+] predicts length of stay and resource uti-lization and that persistent hyponatremia is an independent predictor of mortal-ity and HF hospitalization.3-5 In multiple prognostic models, serum [Na+] predicts outcome.6-8 For example, in the Seattle Heart Failure Model, each decrement in serum [Na+] is associated with decreases in 1-, 2-, and 5-year survival.7

What should be done to acutely mobilize fluid during inpatient care?The options for diuretic therapy include continuous IV loop diuretic, intermittent IV loop diuretic, and IV loop diuretic following pretreatment with a thiazide. However, as shown by Rogers and col-leagues, diuretic therapy results in a sig-nificant decline in the glomerular filtra-tion rate (GFR).9

Recently, the DOSE study compared continuous infusion of furosemide ver-sus q12-hour bolus strategies at both low and high doses in a 2x2 factorial design.10 Low dose was defined by the patient’s oral dose converted to an IV dose. High dose was defined by the oral dose multiplied by a factor of 2.5, both continued for a mini-mum of 48 hours. Patients who received the high-dose formulation were more likely to be congestion-free, had greater loss of weight, and greater net loss of volume. In addition, the reduction in B-type natriuretic peptide levels was greater, though the significance of this finding is not clear. However, there was a clinical consequence with high-dose furosemide therapy: a greater percentage of patients (23% vs 14%) experienced an increase in creatinine of >0.3 mg/dL. Though a crude reflection of GFR, this magnitude of change has often been used as a sign of worsening renal function in clinical studies of HF and is associated with a worse prognosis. The primary end point

case sTudy 2: hyPnoTremia in hearT failure Paul J. hauPTman, md

Page 11: Visit Clinical Perspectives in ... · Clinical Perspectives in Hyponatremia n March 2012 3 The PaThoPhysiology and significance of hyPonaTremia JosePh g. Verbalis, md Clinical Perspectives

Clinical Perspectives in Hyponatremia n March 2012 11

factors, relief of dyspnea, achievement of weight loss, improvement in serum sodium if low and avoidance of worsening renal function, all of which may lessen the risk of readmission (though other nonphysiologic factors may come into play).14

Some of these variables are related; for example, there appears to be a linear relationship between reductions in body weight and dyspnea.15 The development of cardiorenal syndrome is a risk during acute management though the cause may be mul-tifactorial. One such pathway may be in-duced by aggressive diuretic therapy leading to neurohormonal activation, diminished blood flow, decreased renal perfusion, and impaired renal function. Clinically, this can manifest as diuretic resistance; when pres-ent, clinical decision-making becomes more complicated and the patient is likely to ex-perience a longer length of stay, increased morbidity and lessened survival.

Additionally, lack of weight loss has been noted to occur in up to 16% of pa-tients during hospitalization for HF.16 On the basis of these studies, one can construct a scenario that places patients in various risk groups for readmission. The presence of worsening renal function, hyponatre-mia, poor quality of life at baseline, lack of weight loss, multiple comorbidities, and lack of engagement with the health care system are key components of this evaluation.

For Mrs. M, inpatient management had achieved dyspnea relief, weight loss, ab-sence of worsening of renal function, and improvement in serum [Na+]. Although these results offer no guarantee of an ex-tended period out of hospital, at the very least they suggest that a degree of clinical stability has been achieved. It might also have been helpful to gauge the patient’s quality of life using a formal administration of a validated instrument; predictors of persistently poor quality of life following an admission for HF have been published.17

Readmission for HF is the number one readmission DRG in the Medicare population. However, patients are often re-admitted for other causes and there are fre-quently competing risks for mortality due to the high prevalence of co-morbidities.

What is the significance of improvement in [Na+] at discharge in patients with HF? We do not know if the approach used to increase serum [Na+] by the time of

in the DOSE study was patient global as-sessment at 72 hours, and there were no significant differences between the groups. In addition, there were no differences in death, rehospitalization, and ED visits at 60 days, though this study is quite small and may have been underpowered to show a difference. Of particular note, elevated venous pressure is increasingly recognized as a ma-jor contributor to worsening renal function. This is an important observation since right ventricular failure may develop in a signifi-cant proportion of patients with HF and these patients may be at risk due to venous congestion at the level of the kidney. Ultrafiltration is an option as demon-strated in the UNLOAD study,11 though the exact timing of initiation and duration of therapy has not been fully evaluated. Fluid removal is generally isotonic, which will not correct hyponatremia. Rather, con-sideration should be given to aggressive fluid restriction and if that fails, the initia-tion of a vaptan. There are two commer-cially available drugs in this class: conivap-tan, a combined V1a/V2 receptor antagonist available in IV formulation, and tolvaptan, a V2 selective oral agent. The latter has been shown to be effective in reducing body weight and increasing urine output in the setting of HF exacerbation,12,13 but long-term efficacy has not been prospectively examined in the hyponatremic HF cohort.

Clinical courseGiven the patient’s symptoms, presence of fluid overload, and clear worsening of se-rum [Na+], IV furosemide, 5 mg by continu-ous infusion and oral tolvaptan, 15 mg daily were administered. On day 1, urine output was 3.5 L and serum [Na+] increased to 128 mEq/L. On day 2 the dose of tolvap-tan was increased to 30 mg/d, and on day 3 the dose of furosemide was increased to 10 mg/h. On day 5, the patient’s total urine output for her first 5 days of hospitalization was 13.5 L. Serum [Na+] had increased to 134 mEq/L. The patient was noted to be eating well and ambulating normally. Her husband noted that she was filling out a crossword puzzle. Her weight was near her baseline, now at 63 kg.

Management considerationsIn the management of acute decom-pensated HF, there are several impor-tant goals: identification of exacerbating

discharge is relevant in patients with HF. We do know that mortality after hospi-talization is related to the initial and final serum [Na+] values. In a patient popula-tion including patients with HF but also other conditions, the highest mortality was among those who had persistent hyponatremia; however, it is worthwhile to note that patients who acquired hypo-natremia during hospitalization also had increased in-hospital mortality as well as 1- and 5-year mortalities relative to pa-tients whose hyponatremia had resolved by the time of discharge.18

References1. Sipahi I, Carrigan TP, Rowland DY, Stambler BS, Fang JC. Impact of QRS duration on clinical event reduction with cardiac resynchronization therapy: meta-analysis of randomized controlled trials. Arch Intern Med. 2011 Sep 12;171(16):1454-62.2. Hauptman PJ, Mikolajczak P, Mohr CJ, George A, Hoover R, Swindle J, Schnitzler M. Chronic continuous home inotropic therapy in end-stage heart failure. Am Heart J. 2006;152:1096.e1-1096.e8.3. Jao GT, Chiong JR. Hyponatremia in acute decompen-sated heart failure: mechanisms, prognosis, and treat-ment options. Clin Cardiol. 2010;33:666-71.4. Gheorghiade M, Rossi JS, Cotts W, Shin DD, et al. Char-acterization and prognostic value of persistent hypona-tremia in patients with severe heart failure in the ESCAPE trial. Arch Intern Med. 2007;167(18):1998-2005. 5. Gheorghiade M, Abraham WT, Albert NM, et al, on be-half of the OPTIMIZE-HF Investigators and Coordinators. Relationship between admission serum sodium concen-tration and clinical outcomes in patients hospitalized for heart failure: an analysis from the OPTIMIZE-HF registry. Eur Heart J. 2007;28:980-8. 6. Aaronson KD, Schwartz JS, Chen T-Z, Wong K-L, Goin JE, Mancini DM. Development and prospective validation of a clinical index to predict survival in ambulatory pa-tients referred for cardiac transplant evaluation. Circula-tion. 1997;95:2660-7.7. Levy WC, Mozaffarian D, Linker DT, Sutradhar SC, et al. Heart failure: The Seattle Heart Failure Model: prediction of survival in heart failure. Circulation. 2006;113:1424-33.8. Lee DS, Austin PC, Rouleau JL, Liu PP, Naimark D, Tu JV. Predicting mortality among patients hospitalized for heart failure. Derivation and validation of a clinical model. JAMA. 2003;290:2581-7.9. Rogers HL. Marshall J. Bock J. et al. A randomized, controlled trial of the renal effects of ultrafiltration as compared to furosemide in patients with acute decom-pensated heart failure. J Cardiac Fail. 2008;14:1-5.10. Felker GM, Lee KL, Bull DA, Redfield MM, et al. Diuretic strategies in patients with acute decompensated heart failure. N Engl J Med. 2011;364:797-805.11. Costanzo MR, Guglin ME, Saltzberg MT, Jessup ML, Bart BA, Teerlink JR, et al for the UNLOAD Trial Investi-gators. Ultrafiltration versus intravenous diuretics for pa-tients hospitalized for acute decompensated heart failure. J Am Coll Cardiol. 2007;49:675-83.12. Gheorghiade M, Gattis WA, O’Connor CM, Adams Jr KF, Elkayam U et al for the Acute and Chronic Therapeutic Impact of a Vasopressin Antagonist in Congestive Heart Failure (ACTIV in CHF) Investigators. Effects of tolvaptan, a vasopressin antagonist, in patients hospitalized with worsening heart failure. JAMA. 2004;291:1963-71. 13. Gheorghiade M, Konstam MA, Burnett JC Jr, Grinfeld L, Maggioni AP, Swedberg K, Udelson JE, Zannad F, Cook T, Ouyang J, Zimmer C, Orlandi C. Efficacy of Vasopressin Antagonism in Heart Failure Outcome Study With Tolvap-tan (EVEREST) Investigators. Short-term clinical effects of tolvaptan, an oral vasopressin antagonist, in patients hospitalized for heart failure: the EVEREST Clinical Status

Page 12: Visit Clinical Perspectives in ... · Clinical Perspectives in Hyponatremia n March 2012 3 The PaThoPhysiology and significance of hyPonaTremia JosePh g. Verbalis, md Clinical Perspectives

12 Clinical Perspectives in Hyponatremia n March 2012

Clinical Perspectives in Hyponatremia

Mrs. W is a 46-year-old woman who was admitted to the Neuro-critical Care Unit with SAH. She

was assessed as Hunt-Hess grade 3, con-fused and mildly lethargic, but following commands. She had a past medical his-tory of hypertension and was an active tobacco smoker.

On admission, the patient’s blood pres-sure was 170/78 mm Hg; heart rate, 71 bpm. Initial lab findings were normal, including chemistry-7 screen, CBC, and coagulation profile. EKG, however, showed widespread T-wave inversions, suggesting neurogenic cardiac injury. CT showed a thick clot in the left sylvian fissure and more diffuse sub-arachnoid blood throughout.

Initial treatment focused on the emer-gency administration of antifibrinolytic therapy, epsilon amino-caproic acid, at a dose of 1 g/h until 4 hours prior to cerebral angiography. This treatment is increasingly being used to minimize the risk of ultra-early rebleeding prior to definitive aneurysm repair. She was given a phenytoin loading dose and maintenance therapy, which con-tinued to reduce the risk of acute seizures that might precipitate an aneurysm rebleed. Her hypertension was treated with IV ni-cardipine as needed to maintain SBP <160 mm Hg. She was also started on nimodip-ine to reduce the risk of delayed treatment ischemia from vasospasm. She also received a nicotine patch to reduce the risk of the stress related to nicotine withdrawal.

A STAT cerebral angiogram re-vealed a 7-mm anterior communicating artery aneurysm that was surgically re-paired. Postoperatively, despite transcra-nial Doppler accelerations, no symptom-atic vasospasm was detected.

Hyponatremia and neurologic dysfunctionHyponatremia is associated with sub-stantial mortality. In a prospective analy-sis of the frequency, cause, and outcome

of hyponatremia in hospitalized patients, Anderson and colleagues noted a 60-fold higher death rate in hospitalized patients who had a [Na+] <130 mEq/L than in patients without documented hyponatremia.1

To assess the outcome of severe hyponatremia and characterize factors influencing outcome in hospitalized pa-tients, Nzerue conducted a retrospec-tive study of 168 hospitalized patients with severe hyponatremia ([Na+] <115 mEq/L).2 Neurologic symptoms were documented in 53% of patients. Half of the patients with symptomatic hypona-tremia experienced changes in their level of consciousness or their sensorium; they were confused, lethargic, and had slowed psychomotor function. Severe hyponatremia was documented as lead-ing to stupor or coma and reducing sei-zure threshold.

Physiologically, in hyponatremia ab-normal electrical discharges in the brain occur more often. In Nzerue’s series, 20% of patients with hyponatremic encepha-lopathy developed seizures.2 Other less common symptoms including nausea and vomiting may be secondary to high intra-cranial pressure (ICP), which can result from intracellular edema and brain swell-ing. Even in absence of increased ICP, mild hyponatremia has been associated with an increased risk of gait disturbances and falls.

To explore the relationship of acute hyponatremia with neurologic dysfunc-tion, Arieff and colleagues plotted plasma sodium levels against the level of con-sciousness among patients with acute hy-ponatremia that had developed within 24 hours or less.3 The investigation showed that the more drastic the fall in plasma sodium, the larger the change in level of consciousness, with patients who were having seizures or were comatose having the lowest sodium levels.3

Neurologic etiology of hyponatremia CNS conditions with a tendency to de-velop hyponatremia include subarach-noid hemorrhage (SAH), intracerebral hemorrhage (ICH), massive cerebral infarction, severe traumatic brain in-jury, and infections such as meningitis and encephalitis. Pain, stress, drugs, in-creased ICP, and hypovolemic states all can stimulate the release of AVP. In ad-dition, intracranial hemorrhage and sur-gical intervention can interrupt neuronal communication and hormonal feedback mechanisms. Therefore, in many neuro-critical care units, serum [Na+] is taken as another vital sign.

The important mechanisms by which CNS disorders cause hyponatre-mia include SIADH and cerebral salt wasting (CSW). SIADH is defined by the presence of AVP when it is supposed to be fully suppressed, which normally oc-curs when plasma osmolality falls below 280 mOsm/kg H2O. This results in im-paired renal water clearance, increased total body water, and the development of hyponatremia. Triggers of SIADH include drug exposures, pulmonary dis-ease, and CNS disease (see Case 1).

CSW is defined by the development of extracellular (ECF) volume depletion because of an abnormality in renal sodi-um transport in patients with intracranial disease. It can be difficult to differenti-ate CSW from SIADH because both can present with hyponatremia and concen-trated urine with natriuresis. The dis-tinction is important because treatment options differ: isotonic volume repletion is indicated in untreated CSW, while pro-motion of free water clearance is recom-mended in SIADH.

In SAH, inappropriate elevation of AVP despite normovolemia occurs in a majority of patients during the first week. The AVP increase is consistent with a central mechanism resulting from

case sTudy 3: hyPonaTremia in neurocriTical care sTePhan a. mayer, md, fccm

Trials. JAMA. 2007;297:1332-43.14. Amarasingham R, Moore BJ, Tabak YP, et al. An au-tomated model to identify heart failure patients at risk for 30-day readmission or death using electronic medical record data. Med Care. 2010;48:981-88.15. Pang PS, Konstam MA, Krasa HB, Swedberg K, et al.

Effects of tolvaptan on dyspnea relief from the EVEREST trial. Eur Heart J. 2009;30(18):2233-40.16. Gheorghiade M, Filippatos G. Reassessing treatment of acute heart failure syndromes: the ADHERE Registry. Eur Heart J. 2005;7(supplB):B13-19. 17. Allen LA, Gheorghiade M, Reid KJ, et al. Identifying

patients hospitalized with heart failure at risk for unfa-vorable future quality of life. Circ Cardiovasc Qual Out-comes. 2011;4:389-98.18. Waikar SS, Mount DB, Curhan GC. Mortality after hospitalization with mild, moderate, and severe hypona-tremia. Am J Med. 2009;122(9):857-65.

Page 13: Visit Clinical Perspectives in ... · Clinical Perspectives in Hyponatremia n March 2012 3 The PaThoPhysiology and significance of hyPonaTremia JosePh g. Verbalis, md Clinical Perspectives

Clinical Perspectives in Hyponatremia n March 2012 13

very thin, weighing less than 50 kg, and very malnourished. He was lying in bed in a fetal position responding in a non-purposeful manner. His blood pressure was slightly low at 105/70 mm Hg; heart rate, 64 bpm. Clinically he was not jaun-

case sTudy 4: hyPonaTremia in cirrhosis florence Wong, mbbs, md, fracP, frcPc

Mr. M, a 74-year-old male, first presented in June 2005 with cirrhosis from chronic hepati-

tis B infection, ascites, and confusion. His wife reported multiple prior hospi-tal admissions because of his refusal to

eat and because of multiple falls. He had also been coming to the hospital for re-peat large volume paracentesis as a treat-ment of ascites associated with his cir-rhotic condition.

Exam revealed that the patient was

the hemorrhage. In these cases, osmotic regulation of AVP escapes the normal regulatory process. In addition, AVP increases occur as a result of ECF and plasma volume depletion due to natri-uresis. The increased AVP following the SAH leads to free water retention and hyponatremia. In cases of euvolemic hyponatremia following SAH, use of an aquaretic agent minimizes electrolyte ex-cretion associated with natriuresis there-by reducing the contribution of volume contraction to increased AVP.

The patient’s postoperative courseMrs. W had mild hydrocephalus, which was managed with serial lumbar punc-tures. She was treated with a euvolemic volume strategy with 0.9% saline given at 100 mL/h, adjusted to maintain a central venous pressure of 6-8 mm Hg. Isotonic fluid resuscitation is a hallmark of SAH management for the prevention of hypo-volemia. Limiting free water intake miti-gates against brain edema and ICP exac-erbation. Treatment resulted in a mildly positive fluid balance. Throughout her early ICU stay, she remained confused and disoriented, but the lethargy gradu-ally resolved. She remained confused and disoriented, but was less and less lethar-gic from one day to the next.

Step-down careAfter 1 week, on day 9, Mrs. W was trans-ferred to the step-down care unit, the central line was discontinued, she was continued on her normal saline at 100 mL/h through peripheral IV, and then the next day she seemed more confused. Lab findings showed a serum [Na+] level of 126 mEq/L. Though the central ve-nous pressures seemed to indicate eu-volemia, through 10 days of care the patient’s serum [Na+] had gradually fallen to 126 mEq/L from 143 mEq/L despite

cumulative positive 24-hour fluid balance over the same period.

Why is this patient hyponatremic? Because the patient is euvolemic, the cause of the hyponatremia is SIADH. For CSW to be associated with hyponatremia, the condition would have to have been uncor-rected or untreated and there would have been progressive negative sodium and fluid balance from one day to the next leading to a state of hypovolemia. Stated another way, untreated CSW leads to hypovolemic hyponatremia.

CSW causes hypo-osmolar hypona-tremia, but intravascular volume is reduced due to excessively high sodium losses that are not rescued or reversed by fluid therapy. Diringer’s classic study of 19 patients con-cluded that following an acute aneurismal SAH, hypervolemic therapy prevents vol-ume contraction but not hyponatremia.4 Diringer’s group hypothesized that humor-al factors may favor both sodium loss and water retention, and that AVP regulation is disturbed and may contribute to hypo-natremia in this setting. A third of the SAH patients in the study developed hypo-natremia.

Treatment options for Mrs. WThe therapeutic choices for this patient included fluid restriction to <1.5 L/d; salt tablets (NaCl), 2 g/d; fludrocortisone, 0.1 mg tid; IV furosemide; 20% mannitol (an osmotic diuretic that will remove free wa-ter); 2% or 3% hypertonic saline; or tolvap-tan, 15 mg once a day.

Mrs. W was treated with tolvaptan and within 24 hours her serum [Na+] had in-creased from 126 to 130 mEq/L. After 3 days of therapy, the patient’s serum [Na+] levels had corrected to 138 mEq/L. Her mental status at that time was less con-fused. Clinical trials indicate that for every 15 mg of tolvaptan, a 3- to 4-point increase

in serum [Na+] levels can be expected 24 hours later.5 Tolvaptan was discontinued. The patient was monitored for another 24 hours and discharged to rehabilitation to make a full neurologic recovery.

Patient cases such as this support a compelling argument that correction of hyponatremia leads to neurologic improve-ment. Even mild hyponatremia, in the ab-sence of frank symptoms, can cause neu-rocognitive dysfunction and gait disorders as was demonstrated in Renneboog’s study (see page 5).6 Hyponatremia also increases the risk of falls. Kengne’s study (see page 5) showed the incidence of hyponatremia is significantly higher among patients admit-ted to the hospital with falls and fractures than in the control populations.7 This is a very important safety issue for all intensiv-ists and all hospitalists but especially for neurointensivists, because we’re trying to help people recover from brain injuries and get out of bed and on their feet to get on with their rehabilitation. That’s why it is im-portant to diagnose and treat hyponatremia when we encounter it.

References1. Anderson RJ, Chung H-M, Kluge R, Schrier RW. Hy-ponatremia: a prospective analysis of its epidemiology and the pathogenic role of vasopressin. Ann Intern Med. 1985;102:164-8.2. Nzerue CM, Baffoe-Bonnie H, You W, Falana B, Dai S. Predictors of outcome in hospitalized patients with severe hyponatremia. J Natl Med Assoc. 2003;95(5):335-43.3. Arieff AI, Llach F, Massry S. Neurological manifestations and morbidity of hyponatremia: correlation with brain wa-ter and electrolytes. Medicine. 1976;55(2):121-9.4. Diringer MN, Wu KC, Verbalis JG, Hanley DF. Hypervol-emic therapy prevents volume contraction but not hypona-tremia following subarachnoid hemorrhage. Ann Neurol. 1992;31:543-50.5. Berl T, Quittnat-Pelletier F, Verbalis JG, Schrier RW, et al. Oral tolvaptan is safe and effective in chronic hyponatre-mia. J Am Soc Nephrol. 2010;21(4):705-12.6. Renneboog B, Musch W, Vandemergel X, Manto MU, Decaux G. Mild chronic hyponatremia is associated with falls, unsteadiness, and attention deficits. Am J Med. 2006;119(1):71.e1-71.e8.7. Kengne FG, Andres C, Sattar L, Melot C, et al. Mild hy-ponatremia and risk of fracture in the ambulatory elderly. QJM. 2008;101(7):583-8.

Page 14: Visit Clinical Perspectives in ... · Clinical Perspectives in Hyponatremia n March 2012 3 The PaThoPhysiology and significance of hyPonaTremia JosePh g. Verbalis, md Clinical Perspectives

14 Clinical Perspectives in Hyponatremia n March 2012

Clinical Perspectives in Hyponatremia

with a poor prognosis and an increased risk of death in patients with decompensated cirrhosis awaiting liver transplantation.2 Therefore, there is a need to treat hypona-tremia in patients with cirrhosis and ascites.

What are the symptoms of hyponatremia? In a study of 223 consecutive hospital-ized patients with symptomatic hypona-tremia ([Na+] 98–128 mEq/L), 49% were found to have malaise or lethargy; 47% had dizzy spells; 35% reported vomiting; 17% had confusion; 17% were falling; 6% had headaches, and 0.9% had seizures.3 All symptoms resolved when the serum [Na+] was corrected by stopping thiazide diuretics the patients had been taking.

In patients with cirrhosis, however, symptoms of hyponatremia can be indistin-guishable from those of advanced cirrhosis, such as a vague feeling of being unwell. In some patients, hyponatremia is asymptom-atic, and therefore it is the responsibility of treating physicians to check electrolytes regularly to detect hyponatremia.

What is the best treatment for significant hyponatremia? Hyponatremia can be corrected in cases such as Mr. M by changing either the nu-merator by adding more sodium or by sub-tracting the denominator by changing total body water (Figure 3, page 6). However, patients with cirrhosis and therefore por-tal hypertension already have renal sodium and water retention. Adding sodium will in-crease total body sodium and also increase fluid retention, worsening ascites as the presence of portal hypertension will push the excess retained fluid into the peritoneal cavity. Therefore, fluid restriction is pre-ferred to correct excess water over excess sodium. Sodium restriction will help reduce the amount of fluid retained and therefore indirectly help to reduce the ascites.

Mr. M was treated by withholding di-uretic therapy and restriction to 500 mL of fluids a day. Oral sodium intake was restricted to 44 mEq per day, which can be difficult to achieve. Electrolytes were checked daily. Mr. M and his wife were counseled to visit a dietitian for instruction about special low-sodium food items. Basi-cally, he was advised not to consume any-thing preserved or available in tins, jars, or packets. Instructing patients to eat mostly fresh food items is helpful.

diced. I was not able to elicit anything but mild asterixis. Abdominal exam did not reveal a palpable liver; however, there was splenomegaly and gross ascites.

Mr. M’s daily medications included furosemide, 40 mg; spironolactone, 100 mg; nadolol (a nonselective beta-blocker), 20 mg; lamivudine (an antiviral), 100 mg; and a multivitamin tablet.

Laboratory findingsMr. M’s findings were as follows: he-moglobin, 99 g/L; WBC, 3.3 X 109/L; platelets, 85 X 109/L; serum [Na+], 113 mEq/L; serum [K+], 4.2 mEq/L; creati-nine, 1.01 mg/dL; apartate aminotrans-ferase, 54 IU/L; alanine aminotransferase, 39 IU/L; alkaline phosphate, 74 IU/L; bilirubin, 0.54 mg/dL; albumin, 3.2 g/dL; and international normalized ratio (INR), 1.35. His liver enzymes were mildly ab-normal. His liver function (as indicated by his bilirubin, albumin, and INR) also was mildly abnormal.

Pathophysiology of hyponatremia in cirrhosis In patients with cirrhosis, total body so-dium is increased as a result of increased renal sodium retention. As a patient pro-gresses through the cirrhotic process, going from the stage of compensated cirrhosis to the development of ascites, to the stage where ascites is no longer re-sponsive to diuretic therapy, to hepatore-nal syndrome there is a gradual reduction in the serum [Na+] as a result of various hemodynamic changes that occur in cir-rhosis. Hyponatremia occurs because wa-ter retention far exceeds sodium retention.

In a survey conducted on patients with liver cirrhosis and ascites, almost half had serum [Na+] ≤130 mEq/L, indica-tive of hyponatremia.1 In the same study, the presence of hyponatremia was asso-ciated with complications such as the de-velopment of hepatic encephalopathy, the presence of hepatorenal syndrome, and the development of spontaneous bacte-rial peritonitis. In fact, the lower the serum [Na+], the higher the prevalence of these complications. Patients with serum [Na+] <130 mEq/L had the greatest frequency of these complications, but the frequency was also increased in patients with serum [Na+] 131-135 mEq/L.

The presence of hyponatremia (se-rum [Na+] <130 mEq/L) is also associated

Following this regimen, the patient’s serum [Na+] rose to 123 mEq/L. How-ever, he continued to be weak and inter-mittently confused. Hypertonic saline is not recommended in cirrhosis, as this will significantly increase the sodium re-tention, and hence will significantly wors-en the ascites. Demeclocycline is not in-dicated in cirrhosis, as it can induce renal dysfunction related to its effects on the renal kallikrein-kinin system, involved in the maintenance of renal blood flow in cirrhosis and ascites. Although urea can give an osmotic diuresis and improve the serum [Na+], it can also induce hepatic encephalopathy in cirrhosis, and there-fore is not recommended.

Therefore, Mr. M was started on an AVP receptor antagonist and gradually over the following 2 weeks, serum [Na+] levels improved, with associated improve-ment in mental alertness. Mr. M received satavaptan, a V2 receptor antagonist avail-able in a clinical trial at the time.

Why is a vaptan effective? Basically, vaptans block the reabsorption of water in the renal collecting duct sys-tem. When AVP is attached to the V2 re-ceptor at the renal collecting duct on the basolateral space, it sets off a series of biochemical reactions, which ultimately result in the insertion of aquaporin-2 on the luminal side of the principal duct cell (Figure 1, page 3). This allows water to be reabsorbed back into the basolateral space, thereby concentrating the urine. In the presence of a vaptan that competi-tively blocks off the V2 receptor, the se-ries of biochemical reactions can no lon-ger be initiated; therefore, no aquaporin-2 can be inserted onto the luminal side of the principal duct cell and water cannot be reabsorbed. This physiologic process induces the production of very dilute urine. Therefore, with less water being reabsorbed, total body water is reduced, helping to normalize the ratio between total body sodium and total body water, correcting serum sodium levels.

Clinical courseDuring the ensuing 2 years, Mr. M was able to continue therapy with satvaptan and his serum [Na+] remained mostly within a normal range. He had no further hospital admissions after August 2005; his wife reported no further episodes of

Page 15: Visit Clinical Perspectives in ... · Clinical Perspectives in Hyponatremia n March 2012 3 The PaThoPhysiology and significance of hyPonaTremia JosePh g. Verbalis, md Clinical Perspectives

Clinical Perspectives in Hyponatremia n March 2012 15

Posttest

For each question below, please indicate your answer in the space provided on the evaluation form on page 16.

1. Hyponatremia appears to be a marker for more severe underlying disease with a poorer prognosis for

A. Heart failureB. CirrhosisC. Neurologic diseaseD. All of the above

2. The level of water in the body is controlled byA. The pituitary glandB. Water consumptionC. The hormone arginine vasopressin (AVP)D. Aquaporin-2 water channels

3. The syndrome of inappropriate antidiuretic hormone secretion (SIADH) is an example of

A. Excessive urination of water because water cannot be reabsorbed by the kidneyB. Dilutional hyponatremiaC. Appropriately elevated serum AVP levels in relation

to serum osmolalityD. Serum osmolality > 280 mOsm/kg H2O

4. A study in Singapore in 2003 showed that the prevalence of hyponatremia ([Na+] <135 mEq/L) was

A. 28% in hospitalized patientsB. 21% in hospital clinicsC. 7% in outpatient community care clinicsD. All of the above

5. In various studies, elderly patients have been shown to have a higher incidence of hyponatremia than other groups. It has been estimated that

A. The prevalence of hyponatremia in the elderly increases to 50% in chronic care facilities

B. The incidence of hyponatremic episodes in the elderly can be as high as 75% in nursing homes

C. The overall incidence of hyponatremia is 7% in the geriatric population

D. In patients over the age of 65, the mortality rate is three-fold higher for hyponatremic patients than for those without hyponatremia on admission to the hospital

6. The goals of managing patients with hyponatremia in acute decompensated heart failure include

A. Relief of dyspneaB. Weight lossC. Improvement in serum [Na+] (if low) and avoidance

of worsening renal functionD. All of the above

7. The seminal article exploring the relationship between acute hyponatremia and neurologic dysfunction, published by Arieff in 1976, demonstrated that

A. Seizures developed in 20% of patients with hyponatremic encephalopathy

B. Half the patients with symptomatic hyponatremia have changes in their level of consciousness or their sensorium

C. The more drastic the fall in serum [Na+], the greater the change in level of consciousness, with patients who were having seizures or were comatose having the lowest serum sodium levels.

D. Neurologic symptoms were documented in 53% of patients

8. In patients with cirrhosis, symptoms of hyponatremia are clearly distinct from those of advanced cirrhosis

A. TrueB. False

9. Which treatment to address hyponatremia is least likely to be indicated in patients with cirrhosis and ascites?

A. Fluid restrictionB. Hypertonic salineC. Arginine receptor antagonistD. Albumin infusion

10. Fluid restriction, long the mainstay of treating chronic hyponatremia

A. Cannot be sustained because it produces intolerable dehydration and thirstB. Works very rapidlyC. Corrects serum [Na+] by 1 to 2 mEq/L per hour D. Targets the mechanistic cause of dilutional hyponatremia

confusion; in fact, vaptan therapy also helped with ascites management, and Mr. M did not require paracentesis after Sep-tember 2005 when he was living at home and self-caring. However, in August 2008, the satavaptan program was closed by the sponsor. Over the next 2 weeks, Mr. M’s serum [Na+] fell from 134 to 125 mEq/L and his wife noticed that he was becom-

ing weak and lethargic again and uninter-ested in his surroundings. Finally, he died in cardiac failure in August 2009. His fall in serum [Na+] following stopping sat-avaptan is consistent with what has been observed in large clinical trials which also show return of the serum [Na+] to the hyponatremic range with withdrawal of vaptan therapy.

References1. Angeli P, Wong F, Gines P, and the CAPPS Investigators. Hepatology. 2006;44:1535-42.

2. London MC, Cardenas A, Guevara M, Quinto L. et al. MELD score and serum sodium in the prediction of surviv-al of patients with cirrhosis awaiting liver transplantation. Gut. 2007;56(9):1283-90.

3. Chow KM, Ching-Ha Kwan B, Szeto CC. Clinical stud-ies of thiazide-induced hyponatremia. J Natl Med Assoc. 2004;96:1305-8.

Page 16: Visit Clinical Perspectives in ... · Clinical Perspectives in Hyponatremia n March 2012 3 The PaThoPhysiology and significance of hyPonaTremia JosePh g. Verbalis, md Clinical Perspectives

Indicia

Complete this form and fax it to 845-398-5108, or mail it to Paradigm Medical Communications, LLC, 523 Route 303, Orangeburg NY 10962

Please indicate your response using the following scale: 1=Strongly Disagree 2=Disagree 3=Neither Agree nor Disagree 4=Agree 5=Strongly Agree 1. The following objectives of the educational activity were achieved. 1 2 3 4 5 a. Evaluate available therapeutic agents and approaches used in the management of hyponatremia, including clinical benefits and potential adverse events O O O O O b. Develop a treatment plan for patients with hyponatremia O O O O O 2. The activity delivered content in a concise, unbiased manner. O O O O O 3. The activity was free of commercial bias. (if disagree or strongly disagree, please describe in #11) O O O O O 4. The teaching strategy used in this activity (print) was appropriate. O O O O O 5. The content of the activity was relevant to my practice. O O O O O 6. Please indicate your response using the following scale: 1=Not at all confident, 3=Confident; 5=Very Confident

7. What barriers to applying what you have learned do you anticipate? ___________________________________________________________________

8. What challenges are you faced with when managing a patient that you have diagnosed with hyponatremia? ___________________________________

9. What other problem(s) or issue(s) in your practice would you like to see addressed in future CME activities? ___________________________________

10. What type of activity format do you prefer? (check all that apply)

O Live meetings O DVD O Web-based O Print O Other (please specify) _______________________________________________

11. Other comments or suggestions _______________________________________________________________________________________________

12. Indicate your degree: O MD/DO O NP O PA O RN O Other (please specify) ___________________________________________

13. Indicate your specialty:

O Cardiology O Critical Care O Endocrinology O Hepatology O Hospital-BasedMedicine O Nephrology O Other___________

OI have completed the activity and claim ________ credit hours (maximum of 1.5).

Request for Credit (Please Print Clearly)

Name: Degree: Signature:

Address: City: State: Zip:

E-mail: Fax: Phone:

16 Clinical Perspectives in Hyponatremia n March 2012

Paradigm Medical Communications, LLC523 Route 303Orangeburg, NY 10962

Posttest Answers

1. __ 2. __ 3. __ 4. __ 5. __ 6. __ 7. __ 8. __ 9. __ 10. __

Prior to participating in this activity, how confident were you in your ability to do the following:

After participating in this activity, how confident are you now in your ability to do the following:

1 2 3 4 5

O O O O O

Evaluate available therapeutic agents and approaches used to manage hyponatremia, including clinical

benefits and potential adverse events

O O O O ODevelop a treatment plan for patients with hyponatremia

1 2 3 4 5

O O O O O O O O O O

Evaluation Form